GET READY FOR THE AMC 12 WITH AoPS
Learn with outstanding instructors and top-scoring students from around the world in our AMC 12 Problem Series online course.
CHECK SCHEDULE

Difference between revisions of "2010 AMC 12B Problems"

m (Problem 2)
Line 20: Line 20:
 
</asy></center>
 
</asy></center>
  
<math>\textbf{(A)}\ 15 \qquad \textbf{(B)}\ 20 \qquad \textbf{(C)}\ 25 \qquad \textbf{(D)}\ 30 \qquad \textbf{(E)}\ 35</math>
+
<math>\textbf{(A)}\ 22 \qquad \textbf{(B)}\ 24 \qquad \textbf{(C)}\ 26 \qquad \textbf{(D)}\ 28 \qquad \textbf{(E)}\ 30</math>
  
 
[[2010 AMC 12B Problems/Problem 2|Solution]]
 
[[2010 AMC 12B Problems/Problem 2|Solution]]
 +
 +
== Problem 3 ==
 +
A ticket to a school play cost <math>x</math> dollars, where <math>x</math> is a whole number. A group of 9<sup>th</sup> graders buys tickets costing a total of <math>\</math>48<math>, and a group of 10<sup>th</sup> graders buys tickets costing a total of </math>\<math>64</math>. How many values for <math>x</math> are possible?
 +
 +
<math>\textbf{(A)}\ 1 \qquad \textbf{(B)}\ 2 \qquad \textbf{(C)}\ 3 \qquad \textbf{(D)}\ 4 \qquad \textbf{(E)}\ 5</math>
 +
 +
[[2010 AMC 12B Problems/Problem 3|Solution]]
 +
 +
== Problem 4 ==
 +
A month with <math>31</math> days has the same number of Mondays and Wednesdays.How many of the seven days of the week could be the first day of this month?
 +
 +
<math>\textbf{(A)}\ 2 \qquad \textbf{(B)}\ 3 \qquad \textbf{(C)}\ 4 \qquad \textbf{(D)}\ 5 \qquad \textbf{(E)}\ 6</math>
 +
 +
[[2010 AMC 12B Problems/Problem 4|Solution]]
 +
 +
== Problem 5 ==
 +
Lucky Larry's teacher asked him to substitute numbers for <math>a</math>, <math>b</math>, <math>c</math>, <math>d</math>, and <math>e</math> in the expression <math>a-(b-(c-(d+e)))</math> and evaluate the result. Larry ignored the parenthese but added and subtracted correctly and obtained the correct result by coincidence. The number Larry sustitued for <math>a</math>, <math>b</math>, <math>c</math>, and <math>d</math> were <math>1</math>, <math>2</math>, <math>3</math>, and <math>4</math>, respectively. What number did Larry substitude for <math>e</math>?
 +
 +
<math>\textbf{(A)}\ -5 \qquad \textbf{(B)}\ -3 \qquad \textbf{(C)}\ 0 \qquad \textbf{(D)}\ 3 \qquad \textbf{(E)}\ 5</math>
 +
 +
[[2010 AMC 12B Problems/Problem 5|Solution]]
 +
 +
More coming -- 11:58 EDT 4/2/10

Revision as of 11:56, 2 April 2010

Problem 1

Makarla attended two meetings during her $9$-hour work day. The first meeting took $45$ minutes and the second meeting took twice as long. What percent of her work day was spent attending meetings?

$\textbf{(A)}\ 15 \qquad \textbf{(B)}\ 20 \qquad \textbf{(C)}\ 25 \qquad \textbf{(D)}\ 30 \qquad \textbf{(E)}\ 35$

Solution

Problem 2

A big $L$ is formed as shown. What is its area?

[asy] unitsize(4mm); defaultpen(linewidth(.8pt));  draw((0,0)--(5,0)--(5,2)--(2,2)--(2,8)--(0,8)--cycle); label("8",(0,4),W); label("5",(5/2,0),S); label("2",(5,1),E); label("2",(1,8),N); [/asy]

$\textbf{(A)}\ 22 \qquad \textbf{(B)}\ 24 \qquad \textbf{(C)}\ 26 \qquad \textbf{(D)}\ 28 \qquad \textbf{(E)}\ 30$

Solution

Problem 3

A ticket to a school play cost $x$ dollars, where $x$ is a whole number. A group of 9th graders buys tickets costing a total of $$48$, and a group of 10<sup>th</sup> graders buys tickets costing a total of$\$64$. How many values for $x$ are possible?

$\textbf{(A)}\ 1 \qquad \textbf{(B)}\ 2 \qquad \textbf{(C)}\ 3 \qquad \textbf{(D)}\ 4 \qquad \textbf{(E)}\ 5$

Solution

Problem 4

A month with $31$ days has the same number of Mondays and Wednesdays.How many of the seven days of the week could be the first day of this month?

$\textbf{(A)}\ 2 \qquad \textbf{(B)}\ 3 \qquad \textbf{(C)}\ 4 \qquad \textbf{(D)}\ 5 \qquad \textbf{(E)}\ 6$

Solution

Problem 5

Lucky Larry's teacher asked him to substitute numbers for $a$, $b$, $c$, $d$, and $e$ in the expression $a-(b-(c-(d+e)))$ and evaluate the result. Larry ignored the parenthese but added and subtracted correctly and obtained the correct result by coincidence. The number Larry sustitued for $a$, $b$, $c$, and $d$ were $1$, $2$, $3$, and $4$, respectively. What number did Larry substitude for $e$?

$\textbf{(A)}\ -5 \qquad \textbf{(B)}\ -3 \qquad \textbf{(C)}\ 0 \qquad \textbf{(D)}\ 3 \qquad \textbf{(E)}\ 5$

Solution

More coming -- 11:58 EDT 4/2/10